Sum of positive elements divided by their “weighted” product - inequality












2














I have following expression,



$$ frac{sum_{i=1}^n x_i}{prod_{i=1}^nx_i^{p_i}} $$



where $p_i$s satisfy $sum p_i = 1$ and $p_i in [0,1]$ and $x_igeq0$, $forall i in 1dots n$.



I think that this expression is always $geq 1$, however, I don't know how to prove it.



Is there anything I can conclude?



Thanks.










share|cite|improve this question




















  • 3




    The weighted AM-GM inequality yields $prod_i x_i^{p_i} leq left(sum_i p_i x_iright)^{sum_i p_i} = left(sum_i p_i x_iright)^1 = sum_i p_i x_i leq sum_i 1 x_i = sum_i x_i$.
    – darij grinberg
    4 hours ago










  • @darijgrinberg That comment could be made an answer. [Anyway I'd upvote it.]
    – coffeemath
    4 hours ago










  • @coffeemath: Good idea.
    – darij grinberg
    3 hours ago
















2














I have following expression,



$$ frac{sum_{i=1}^n x_i}{prod_{i=1}^nx_i^{p_i}} $$



where $p_i$s satisfy $sum p_i = 1$ and $p_i in [0,1]$ and $x_igeq0$, $forall i in 1dots n$.



I think that this expression is always $geq 1$, however, I don't know how to prove it.



Is there anything I can conclude?



Thanks.










share|cite|improve this question




















  • 3




    The weighted AM-GM inequality yields $prod_i x_i^{p_i} leq left(sum_i p_i x_iright)^{sum_i p_i} = left(sum_i p_i x_iright)^1 = sum_i p_i x_i leq sum_i 1 x_i = sum_i x_i$.
    – darij grinberg
    4 hours ago










  • @darijgrinberg That comment could be made an answer. [Anyway I'd upvote it.]
    – coffeemath
    4 hours ago










  • @coffeemath: Good idea.
    – darij grinberg
    3 hours ago














2












2








2


1





I have following expression,



$$ frac{sum_{i=1}^n x_i}{prod_{i=1}^nx_i^{p_i}} $$



where $p_i$s satisfy $sum p_i = 1$ and $p_i in [0,1]$ and $x_igeq0$, $forall i in 1dots n$.



I think that this expression is always $geq 1$, however, I don't know how to prove it.



Is there anything I can conclude?



Thanks.










share|cite|improve this question















I have following expression,



$$ frac{sum_{i=1}^n x_i}{prod_{i=1}^nx_i^{p_i}} $$



where $p_i$s satisfy $sum p_i = 1$ and $p_i in [0,1]$ and $x_igeq0$, $forall i in 1dots n$.



I think that this expression is always $geq 1$, however, I don't know how to prove it.



Is there anything I can conclude?



Thanks.







calculus inequality summation products






share|cite|improve this question















share|cite|improve this question













share|cite|improve this question




share|cite|improve this question








edited 3 hours ago









darij grinberg

10.4k33062




10.4k33062










asked 4 hours ago









Michael MarkMichael Mark

1119




1119








  • 3




    The weighted AM-GM inequality yields $prod_i x_i^{p_i} leq left(sum_i p_i x_iright)^{sum_i p_i} = left(sum_i p_i x_iright)^1 = sum_i p_i x_i leq sum_i 1 x_i = sum_i x_i$.
    – darij grinberg
    4 hours ago










  • @darijgrinberg That comment could be made an answer. [Anyway I'd upvote it.]
    – coffeemath
    4 hours ago










  • @coffeemath: Good idea.
    – darij grinberg
    3 hours ago














  • 3




    The weighted AM-GM inequality yields $prod_i x_i^{p_i} leq left(sum_i p_i x_iright)^{sum_i p_i} = left(sum_i p_i x_iright)^1 = sum_i p_i x_i leq sum_i 1 x_i = sum_i x_i$.
    – darij grinberg
    4 hours ago










  • @darijgrinberg That comment could be made an answer. [Anyway I'd upvote it.]
    – coffeemath
    4 hours ago










  • @coffeemath: Good idea.
    – darij grinberg
    3 hours ago








3




3




The weighted AM-GM inequality yields $prod_i x_i^{p_i} leq left(sum_i p_i x_iright)^{sum_i p_i} = left(sum_i p_i x_iright)^1 = sum_i p_i x_i leq sum_i 1 x_i = sum_i x_i$.
– darij grinberg
4 hours ago




The weighted AM-GM inequality yields $prod_i x_i^{p_i} leq left(sum_i p_i x_iright)^{sum_i p_i} = left(sum_i p_i x_iright)^1 = sum_i p_i x_i leq sum_i 1 x_i = sum_i x_i$.
– darij grinberg
4 hours ago












@darijgrinberg That comment could be made an answer. [Anyway I'd upvote it.]
– coffeemath
4 hours ago




@darijgrinberg That comment could be made an answer. [Anyway I'd upvote it.]
– coffeemath
4 hours ago












@coffeemath: Good idea.
– darij grinberg
3 hours ago




@coffeemath: Good idea.
– darij grinberg
3 hours ago










2 Answers
2






active

oldest

votes


















3














Applying the weighted AM-GM inequality to the weights $p_1, p_2, ldots, p_n$, we obtain
begin{equation}
dfrac{p_1 x_1 + p_2 x_2 + cdots + p_n x_n}{1} geq sqrt[1]{x_1^{p_1} x_2^{p_2} cdots x_n^{p_n}}
end{equation}

(since $p_1 + p_2 + cdots + p_n = sum_{i=1}^n p_i = 1$). This simplifies to
begin{equation}
p_1 x_1 + p_2 x_2 + cdots + p_n x_n geq x_1^{p_1} x_2^{p_2} cdots x_n^{p_n} .
end{equation}

Hence,
begin{align}
x_1^{p_1} x_2^{p_2} cdots x_n^{p_n}
leq p_1 x_1 + p_2 x_2 + cdots + p_n x_n
= sum_{i=1}^n underbrace{p_i}_{substack{leq 1 \ text{(since $p_i in left[0,1right]$)}}} x_i leq sum_{i=1}^n x_i ,
end{align}

so that
begin{align}
sum_{i=1}^n x_i leq x_1^{p_1} x_2^{p_2} cdots x_n^{p_n} = prod_{i=1}^n x_i^{p_i} .
end{align}






share|cite|improve this answer





























    4














    Concavity of $log$ gives
    $$logleft( prod_{i=1}^nx_i^{p_i}right) = sum_{i=1}^n p_ilog x_i stackrel{mbox{concavity}}{leq} logleft( sum_{i=1}^np_i x_iright) stackrel{0leq p_ileq 1}{leq} logleft( sum_{i=1}^nx_iright)$$






    share|cite|improve this answer





















      Your Answer





      StackExchange.ifUsing("editor", function () {
      return StackExchange.using("mathjaxEditing", function () {
      StackExchange.MarkdownEditor.creationCallbacks.add(function (editor, postfix) {
      StackExchange.mathjaxEditing.prepareWmdForMathJax(editor, postfix, [["$", "$"], ["\\(","\\)"]]);
      });
      });
      }, "mathjax-editing");

      StackExchange.ready(function() {
      var channelOptions = {
      tags: "".split(" "),
      id: "69"
      };
      initTagRenderer("".split(" "), "".split(" "), channelOptions);

      StackExchange.using("externalEditor", function() {
      // Have to fire editor after snippets, if snippets enabled
      if (StackExchange.settings.snippets.snippetsEnabled) {
      StackExchange.using("snippets", function() {
      createEditor();
      });
      }
      else {
      createEditor();
      }
      });

      function createEditor() {
      StackExchange.prepareEditor({
      heartbeatType: 'answer',
      autoActivateHeartbeat: false,
      convertImagesToLinks: true,
      noModals: true,
      showLowRepImageUploadWarning: true,
      reputationToPostImages: 10,
      bindNavPrevention: true,
      postfix: "",
      imageUploader: {
      brandingHtml: "Powered by u003ca class="icon-imgur-white" href="https://imgur.com/"u003eu003c/au003e",
      contentPolicyHtml: "User contributions licensed under u003ca href="https://creativecommons.org/licenses/by-sa/3.0/"u003ecc by-sa 3.0 with attribution requiredu003c/au003e u003ca href="https://stackoverflow.com/legal/content-policy"u003e(content policy)u003c/au003e",
      allowUrls: true
      },
      noCode: true, onDemand: true,
      discardSelector: ".discard-answer"
      ,immediatelyShowMarkdownHelp:true
      });


      }
      });














      draft saved

      draft discarded


















      StackExchange.ready(
      function () {
      StackExchange.openid.initPostLogin('.new-post-login', 'https%3a%2f%2fmath.stackexchange.com%2fquestions%2f3068593%2fsum-of-positive-elements-divided-by-their-weighted-product-inequality%23new-answer', 'question_page');
      }
      );

      Post as a guest















      Required, but never shown

























      2 Answers
      2






      active

      oldest

      votes








      2 Answers
      2






      active

      oldest

      votes









      active

      oldest

      votes






      active

      oldest

      votes









      3














      Applying the weighted AM-GM inequality to the weights $p_1, p_2, ldots, p_n$, we obtain
      begin{equation}
      dfrac{p_1 x_1 + p_2 x_2 + cdots + p_n x_n}{1} geq sqrt[1]{x_1^{p_1} x_2^{p_2} cdots x_n^{p_n}}
      end{equation}

      (since $p_1 + p_2 + cdots + p_n = sum_{i=1}^n p_i = 1$). This simplifies to
      begin{equation}
      p_1 x_1 + p_2 x_2 + cdots + p_n x_n geq x_1^{p_1} x_2^{p_2} cdots x_n^{p_n} .
      end{equation}

      Hence,
      begin{align}
      x_1^{p_1} x_2^{p_2} cdots x_n^{p_n}
      leq p_1 x_1 + p_2 x_2 + cdots + p_n x_n
      = sum_{i=1}^n underbrace{p_i}_{substack{leq 1 \ text{(since $p_i in left[0,1right]$)}}} x_i leq sum_{i=1}^n x_i ,
      end{align}

      so that
      begin{align}
      sum_{i=1}^n x_i leq x_1^{p_1} x_2^{p_2} cdots x_n^{p_n} = prod_{i=1}^n x_i^{p_i} .
      end{align}






      share|cite|improve this answer


























        3














        Applying the weighted AM-GM inequality to the weights $p_1, p_2, ldots, p_n$, we obtain
        begin{equation}
        dfrac{p_1 x_1 + p_2 x_2 + cdots + p_n x_n}{1} geq sqrt[1]{x_1^{p_1} x_2^{p_2} cdots x_n^{p_n}}
        end{equation}

        (since $p_1 + p_2 + cdots + p_n = sum_{i=1}^n p_i = 1$). This simplifies to
        begin{equation}
        p_1 x_1 + p_2 x_2 + cdots + p_n x_n geq x_1^{p_1} x_2^{p_2} cdots x_n^{p_n} .
        end{equation}

        Hence,
        begin{align}
        x_1^{p_1} x_2^{p_2} cdots x_n^{p_n}
        leq p_1 x_1 + p_2 x_2 + cdots + p_n x_n
        = sum_{i=1}^n underbrace{p_i}_{substack{leq 1 \ text{(since $p_i in left[0,1right]$)}}} x_i leq sum_{i=1}^n x_i ,
        end{align}

        so that
        begin{align}
        sum_{i=1}^n x_i leq x_1^{p_1} x_2^{p_2} cdots x_n^{p_n} = prod_{i=1}^n x_i^{p_i} .
        end{align}






        share|cite|improve this answer
























          3












          3








          3






          Applying the weighted AM-GM inequality to the weights $p_1, p_2, ldots, p_n$, we obtain
          begin{equation}
          dfrac{p_1 x_1 + p_2 x_2 + cdots + p_n x_n}{1} geq sqrt[1]{x_1^{p_1} x_2^{p_2} cdots x_n^{p_n}}
          end{equation}

          (since $p_1 + p_2 + cdots + p_n = sum_{i=1}^n p_i = 1$). This simplifies to
          begin{equation}
          p_1 x_1 + p_2 x_2 + cdots + p_n x_n geq x_1^{p_1} x_2^{p_2} cdots x_n^{p_n} .
          end{equation}

          Hence,
          begin{align}
          x_1^{p_1} x_2^{p_2} cdots x_n^{p_n}
          leq p_1 x_1 + p_2 x_2 + cdots + p_n x_n
          = sum_{i=1}^n underbrace{p_i}_{substack{leq 1 \ text{(since $p_i in left[0,1right]$)}}} x_i leq sum_{i=1}^n x_i ,
          end{align}

          so that
          begin{align}
          sum_{i=1}^n x_i leq x_1^{p_1} x_2^{p_2} cdots x_n^{p_n} = prod_{i=1}^n x_i^{p_i} .
          end{align}






          share|cite|improve this answer












          Applying the weighted AM-GM inequality to the weights $p_1, p_2, ldots, p_n$, we obtain
          begin{equation}
          dfrac{p_1 x_1 + p_2 x_2 + cdots + p_n x_n}{1} geq sqrt[1]{x_1^{p_1} x_2^{p_2} cdots x_n^{p_n}}
          end{equation}

          (since $p_1 + p_2 + cdots + p_n = sum_{i=1}^n p_i = 1$). This simplifies to
          begin{equation}
          p_1 x_1 + p_2 x_2 + cdots + p_n x_n geq x_1^{p_1} x_2^{p_2} cdots x_n^{p_n} .
          end{equation}

          Hence,
          begin{align}
          x_1^{p_1} x_2^{p_2} cdots x_n^{p_n}
          leq p_1 x_1 + p_2 x_2 + cdots + p_n x_n
          = sum_{i=1}^n underbrace{p_i}_{substack{leq 1 \ text{(since $p_i in left[0,1right]$)}}} x_i leq sum_{i=1}^n x_i ,
          end{align}

          so that
          begin{align}
          sum_{i=1}^n x_i leq x_1^{p_1} x_2^{p_2} cdots x_n^{p_n} = prod_{i=1}^n x_i^{p_i} .
          end{align}







          share|cite|improve this answer












          share|cite|improve this answer



          share|cite|improve this answer










          answered 3 hours ago









          darij grinbergdarij grinberg

          10.4k33062




          10.4k33062























              4














              Concavity of $log$ gives
              $$logleft( prod_{i=1}^nx_i^{p_i}right) = sum_{i=1}^n p_ilog x_i stackrel{mbox{concavity}}{leq} logleft( sum_{i=1}^np_i x_iright) stackrel{0leq p_ileq 1}{leq} logleft( sum_{i=1}^nx_iright)$$






              share|cite|improve this answer


























                4














                Concavity of $log$ gives
                $$logleft( prod_{i=1}^nx_i^{p_i}right) = sum_{i=1}^n p_ilog x_i stackrel{mbox{concavity}}{leq} logleft( sum_{i=1}^np_i x_iright) stackrel{0leq p_ileq 1}{leq} logleft( sum_{i=1}^nx_iright)$$






                share|cite|improve this answer
























                  4












                  4








                  4






                  Concavity of $log$ gives
                  $$logleft( prod_{i=1}^nx_i^{p_i}right) = sum_{i=1}^n p_ilog x_i stackrel{mbox{concavity}}{leq} logleft( sum_{i=1}^np_i x_iright) stackrel{0leq p_ileq 1}{leq} logleft( sum_{i=1}^nx_iright)$$






                  share|cite|improve this answer












                  Concavity of $log$ gives
                  $$logleft( prod_{i=1}^nx_i^{p_i}right) = sum_{i=1}^n p_ilog x_i stackrel{mbox{concavity}}{leq} logleft( sum_{i=1}^np_i x_iright) stackrel{0leq p_ileq 1}{leq} logleft( sum_{i=1}^nx_iright)$$







                  share|cite|improve this answer












                  share|cite|improve this answer



                  share|cite|improve this answer










                  answered 3 hours ago









                  trancelocationtrancelocation

                  9,5601622




                  9,5601622






























                      draft saved

                      draft discarded




















































                      Thanks for contributing an answer to Mathematics Stack Exchange!


                      • Please be sure to answer the question. Provide details and share your research!

                      But avoid



                      • Asking for help, clarification, or responding to other answers.

                      • Making statements based on opinion; back them up with references or personal experience.


                      Use MathJax to format equations. MathJax reference.


                      To learn more, see our tips on writing great answers.




                      draft saved


                      draft discarded














                      StackExchange.ready(
                      function () {
                      StackExchange.openid.initPostLogin('.new-post-login', 'https%3a%2f%2fmath.stackexchange.com%2fquestions%2f3068593%2fsum-of-positive-elements-divided-by-their-weighted-product-inequality%23new-answer', 'question_page');
                      }
                      );

                      Post as a guest















                      Required, but never shown





















































                      Required, but never shown














                      Required, but never shown












                      Required, but never shown







                      Required, but never shown

































                      Required, but never shown














                      Required, but never shown












                      Required, but never shown







                      Required, but never shown







                      Popular posts from this blog

                      404 Error Contact Form 7 ajax form submitting

                      How to know if a Active Directory user can login interactively

                      TypeError: fit_transform() missing 1 required positional argument: 'X'